Loophole Question Stem Types

Pataasin ang iyong marka sa homework at exams ngayon gamit ang Quizwiz!

The answer to which of the following questions is most relevant to evaluating the conclusion drawn above?

Evaluate (Powerful)

Which of the following would be most helpful to determine whether A's argument could be logically defended against B's objection?

Evaluate (Powerful)

Which of the following would be most helpful to know in order to judge whether ___ calls into question the hypothesis?

Evaluate (Powerful)

Which one of the following would be most important to know in evaluating the hypothesis in the passage?

Evaluate (Powerful)

Which one of the following would be most relevant to investigate in evaluating the conclusion of the argument?

Evaluate (Powerful)

Which of the following is an error of reasoning in A's argument?

Classic Flaw (Provable)

Which of the following most accurately describes a flaw in the reasoning of the argument?

Classic Flaw (Provable)

The main point of the argument is that:

Conclusion (Provable)

Which of the following most accurately expresses the main conclusion of the argument?

Conclusion (Provable)

Which of the following most accurately expresses the overall conclusion drawn in the argument?

Conclusion (Provable)

Assuming the author's statements are accurate, which one of the following statements CANNOT be true?

Contradiction (Powerful)

If the statements above are true, each of the following could be true EXCEPT:

Contradiction (Powerful)

If the statements above are true, then on the basis of them, which one of the following cannot be true?

Contradiction (Powerful)

The information above, if accurate, can best be used as evidence against which one of the following hypotheses?

Contradiction (Powerful)

A and B disagree over whether:

Controversy (Provable)

An issue in dispute between A and B is:

Controversy (Provable)

The exchange between A and B most strongly supports the view that they disagree as to:

Controversy (Provable)

The point at issue between A and B is whether:

Controversy (Provable)

The statements above provide the most support for holding that A would disagree with B about which of the following?

Controversy (Provable)

Which of the following is the point at issue between A and B?

Controversy (Provable)

Which of the following, if true, is the strongest counter A can make to B's objection?

Counter (Powerful)

Which of the following, if true, would provide A with the strongest counter to B's response?

Counter (Powerful)

Which of the following principles, if valid, most strongly supports the reasoning above?

Strengthen (Powerful)

Which of the following can be properly concluded/inferred from the info given above?

Inference (Provable)

Which of the following follows logically from the statements above?

Inference (Provable)

Which of the following must be true on the basis of the information above?

Inference (Provable)

A's reasoning is flawed because A presumes without giving sufficient justification that:

Loophole Flaw (Provable)

The reasoning in the argument is questionable in that it overlooks/ignores the possibility that:

Loophole Flaw (Provable)

The reasoning is most vulnerable to criticism on the grounds that it fails to consider whether:

Loophole Flaw (Provable)

A responds to B's argument by:

Method (Provable)

The author attempts to persuade by doing all of the following EXCEPT:

Method (Provable)

The relationship of A's response to B's argument is that A's response:

Method (Provable)

This argument uses which of the following argumentative techniques?

Method (Provable)

Which most accurately describes A's strategy of argumentation?

Method (Provable)

Which of the following most accurately describes how the argument proceeds?

Method (Provable)

The information above provides the most support for which one of the following statements?

Most Strongly Supported (Provable)

The statements above most strongly support the conclusion that _______

Most Strongly Supported (Provable)

The statements above, if true, most strongly support which of the following?

Most Strongly Supported (Provable)

Which of the following is most strongly supported by the info above?

Most Strongly Supported (Provable)

Which of the following, if true, most helps to strengthen the argument?

Strengthen (Powerful)

The conclusion drawn above follows logically if which one of the following is assumed?

Sufficient Assumption (Powerful)

Which one of the following is an assumption that would allow the conclusion to be properly drawn?

Sufficient Assumption (Powerful)

The argument assumes which one of the following?

Necessary Assumption (Provable)

The argument depends on assuming that:

Necessary Assumption (Provable)

The reasoning in the argument depends on that:

Necessary Assumption (Provable)

Which of the following is an assumption on which the argument relies?

Necessary Assumption (Provable)

Which of the following is an assumption required by the argument?

Necessary Assumption (Provable)

The flawed reasoning in which of the following arguments most closely resembles the flawed reasoning in the argument above?

Parallel Flaw (Provable)

Which of the following is most appropriate as an analogy demonstrating that the reasoning in the argument above is flawed?

Parallel Flaw (Provable)

Which one of the following arguments exhibits flawed reasoning that is most parallel to that in the argument above?

Parallel Flaw (Provable)

In terms of its logical features, the argument above most resembles which of the following?

Parallel Reasoning (Provable)

The pattern of reasoning in the argument above is most closely paralleled in which of the following?

Parallel Reasoning (Provable)

The pattern of reasoning in the argument above is most similar to that in which of the following?

Parallel Reasoning (Provable)

The reasoning in which of the following is most similar to the reasoning above?

Parallel Reasoning (Provable)

The reasoning above conforms most closely to which of the following propositions?

Principle Conform (Provable)

Which of the following conforms most closely to the principle illustrated above?

Principle Conform (Provable)

Which of the following propositions is best illustrated by the passage?

Principle Conform (Provable)

Which of the following propositions is best illustrated by the situation described in the passage?

Principle Conform (Provable)

Which of the following principles, if valid, most helps to justify the reasoning in the argument?

Strengthen (Powerful)

Each of the following describes a flaw in A's reasoning EXCEPT:

Classic Flaw (Provable)

The reasoning in the argument is most vulnerable to criticism on the grounds that the argument:

Classic Flaw (Provable)

Which of the following is a questionable technique employed by A in responding to B?

Classic Flaw (Provable)

If A and B are both sincere in what they say, then it can be properly concluded that they agree that:

Agreement (Provable)

On the basis of their statements, A and B are committed to agreeing about which of the following?

Agreement (Provable)

The reference to "_____" plays which of the following roles in the argument?

Argument Part (Provable)

The statement/phrase that "____" serves which of the following functions in the argument?

Argument Part (Provable)

The conclusion of the argument is strongly supported if which one of the following completes the argument?

Fill In (Provable)

Which of the following most logically completes the argument?

Fill In (Provable)

A consequence of the view above is that:

Inference (Provable)

If the statements above are true, which one of the following is an inference that can be properly drawn on the basis of them?

Inference (Provable)

If the statements above are true, which one of the following must also be true?

Inference (Provable)

Which of the following, if true, does most to explain the surprising result?

Resolution (Powerful)

Which of the following, if true, helps to resolve the apparent conflict described above?

Resolution (Powerful)

Which of the following, if true, most helps to resolve the apparent discrepancy in the information above?

Resolution (Powerful)

Which of the following, if true, most helps to resolve the apparent paradox?

Resolution (Powerful)

Which of the following, if true, most clearly casts doubt on that implication?

Weaken (Powerful)

Which of the following, if true, most tends to undermine the argument?

Weaken (Powerful)

Which one of the following, if true, most seriously weakens the argument above?

Weaken (Powerful)


Kaugnay na mga set ng pag-aaral

Exam 2 Geriatrics: Prevention & Health Maintenance

View Set

EXAM 2 CHs (#24,25,26) Review Guide

View Set

Ch. 20 Accounting Changes and Error Corrections

View Set

Unit 2 (Reformation and the Age of Exploration)

View Set

Rhomboid Major & Minor, Trapezius og Latissimus Dorsi

View Set